Đến nội dung

Mashimaru nội dung

Có 218 mục bởi Mashimaru (Tìm giới hạn từ 28-04-2020)



Sắp theo                Sắp xếp  

#224400 Lớp luyện thi VMO 2010 trên mạng

Đã gửi bởi Mashimaru on 31-12-2009 - 18:35 trong Seminar Phương pháp toán sơ cấp

Thưa thầy, xin thầy xem lại bài 1 giúp em ạ. Bất đẳng thức sai tại $x=3, y=-6,z=7$.



#224116 Đăng ký tham gia ban tổ chức VMEO IV

Đã gửi bởi Mashimaru on 29-12-2009 - 21:32 trong Thông báo tổng quan

Em xin đăng ký ạ.

Họ tên: Phạm Hy Hiếu
Nick trong diễn đàn toán học 1.0: Mashimaru
Năm sinh: 1992
Quê quán: Vĩnh Phúc
Nơi ở hiện tại: Thành phố Hồ Chí Minh
Số điện thoại: 0126 9500 218
Nick Yahoo: [email protected]
Nick Skype: Không có
Hòm thư: [email protected]
Nghề nghiệp: Học sinh.


Em xin đăng ký tham gia phần 2. và 3. ạ:
2. Tham gia quảng bá cuộc thi: em có thể quảng bá cuộc thi bằng cách giới thiệu nó với các bạn trong lớp và các khóa dưới ở trường.
3. Tham gia dự thi.



#221468 Đa thức

Đã gửi bởi Mashimaru on 25-11-2009 - 01:42 trong Phương trình - Hệ phương trình - Bất phương trình

Đặt $f(x) = \prod_{i=1}^{k} (x-a_i)^{r_i}$ với $a_1 < a_2 < ... < a_k$ và $\sum_{i=1}^{k} r_i =n$. Theo định lý Rolle, đa thức đạo hàm $f'(x)$ đều nhận các nghiệm $a_i$ với bội $r_i-1$, đồng thời trong mỗi khoảng $(a_i,a_{i+1})$ thì $f'(x)$ còn có ít nhất một nghiệm khác. Như vậy, số nghiệm của $f'(x)$ được liệt kê đã là $\sum_{i=1}^{k} (r_i - 1) + (k-1)= (n-k) + (k-1) = n-1$. Nhưng $deg f'(x) = n-1$ nên các nghiệm trong các khoảng $(a_i,a_{i+1})$ của $f'(x)$ không thể là nghiệm bội. Do đó, nghiệm bội của $f'(x)$ nếu có phải là một trong số các $a_i$ và ta có đpcm.



#221467 Đề chọn đội tuyển toán PTNK 2009

Đã gửi bởi Mashimaru on 25-11-2009 - 01:34 trong Thi HSG cấp Tỉnh, Thành phố. Olympic 30-4. Đề thi và kiểm tra đội tuyển các cấp.

Đây là lời giải của em. Mọi người xem thử ạ.

File gửi kèm




#210971 Một tính chất của đường đối trung

Đã gửi bởi Mashimaru on 19-08-2009 - 20:22 trong Hình học

Cách tiếp cận của mình là như sau:

Gọi $d_a$ là đường trung trực của $BC$ và $A_2$ là cực của $BC$ đối với $(O)$. Thế thì $O,O_a,A_2\in d_a$. Qua $A$ lại kẻ đường thẳng $l_a // OL$, đặt $S_a\equiv l_a\cap d_a$. Để chứng minh $AO_a$ đi qua trung điểm của $OL$ thì ta cần chứng minh $(AA_2,AO,AO_a,AS_a)=-1$ hay $(A_2OO_aS_a)=-1$. Lúc này chiếu $O_a,O,A_2$ lên $AB$, với chú rằng $(OO_a,AB)\equiv \dfrac{\pi}{2}+(BC,AB)\pmod{\pi}$, ta tính được tỉ số kép $(A_2OO_aS_a)=-1$.

Ngoài ra nếu đề chỉ yêu cầu là "chứng minh $AO_a,BO_b,CO_c$ đồng qui" thì có một lời giải khá đẹp bằng nghịch đảo. Tuy nhiên để chỉ ra tính chất của điểm đồng qui thì mình chưa tìm được cách nào ít phải tính toán hơn cách này.



#210968 Tập hợp

Đã gửi bởi Mashimaru on 19-08-2009 - 20:02 trong Tổ hợp và rời rạc

Nhìn quen quen, IRAN năm nào đó thì phải :supset.


Đề thi đội dự tuyển trường em ạ :supset



#210899 Ánh xạ

Đã gửi bởi Mashimaru on 18-08-2009 - 23:57 trong Tổ hợp và rời rạc

Các tập hợp $S,T$ có phải thỏa điều kiện gì và ánh xạ có cần điều kiện gì không Trung? Nếu không thì cho mọi điểm thuộc $S$ thành đúng một điểm thuộc $T$ là xong mà?



#210898 Một bài hay...

Đã gửi bởi Mashimaru on 18-08-2009 - 23:52 trong Các bài toán Đại số khác

Ừ nhỉ, tớ không để ý profile của cậu bên trái, hóa ra là đồng hương với nhau cả ^^. Tớ cũng vừa hỏi thầy tớ rồi, có lẽ đa thức đồng biến hiểu như vậy thôi. ilovemath1809 học Lê Hồng Phong à?



#210895 TST Nghệ An

Đã gửi bởi Mashimaru on 18-08-2009 - 23:36 trong Tổ hợp và rời rạc

Anh tanlsth ơi, em thấy thực ra điều kiện để phương trình $ax+by=n$ với $(a,b)=1$ có nghiệm nguyên dương thì chỉ cần $n > ab - a -b$ thôi ạ, tức là mạnh hơn điều kiện anh đưa ra. Nhưng $t_i$ anh nói đến là gì ạ? Có phải là "phần tử lớn thứ $i$ của $T$" không ạ? Mà nếu là như thế thì làm sao chứng minh $t_i \leq 2i-1$ được ạ?



#210893 Tập hợp

Đã gửi bởi Mashimaru on 18-08-2009 - 23:24 trong Tổ hợp và rời rạc

Giả sử ngược lại, ta xét các tập con $A_1,A_2,...,A_k$ của $S=\{1,2,...,n\}$ sao cho $|A_i_1\cap A_i_2 \cap A_i_3 \cap A_i_4| \leq n-2$ với mọi $1\leq i_1 < i_2 < i_3 < i_4 \leq k$. Ta sẽ chứng minh rằng $k\leq 2^{n-2}$.
Thật vậy, ta gọi một tập hợp $H\subset S$ là đẹp nếu tồn tại $i,j$ sao cho $1 \leq i \leq j \leq k$ và $H \subset A_i\cup A_j$. Bây giờ xét $H$ là tập con đẹp của $S$ có số phần tử ít nhất. Thế thì phải tồn tại $i,j$ sao cho $1\leq i < j \leq k$ và $H=A_i\cup A_j - \{m\}$ với $m\in S$ nào đó.
Xét $H^* = \{H\cap A_i | 1\leq i \leq k\}$. Ta sẽ chứng minh rằng với mọi tập hợp $H'\subset H$, nếu $H'\in H^*$ thì $H-H'\not \in H^*$. Thật vậy, giả sử ngược lại, thế thì tồn tại $i,j$ sao cho $H'=H\cap A_i$, $H - H' = H\cap A_j$, do đó $H=(H\cap A_i)\cup(H\cap A_j)\subset A_i\cup A_j$, mâu thuẫn với định nghĩa của $H$. Mặt khác, $H$ có $2^{|H|}$ tập con nên $|H^*|\leq 2^{|H|-1}$.
Lại xét tập hợp $T=S - H$ và đặt $T^* = \{T\cap A_i | i=1,2,...,k\}$. Ta cũng sẽ chứng minh điều tương tự với $T^*$. Thật vậy, giả sử tồn tại $T'\subset T$ sao cho $T',T-T'\in T^*$. Ta có $T'=T\cap A_k$ và $T - T' = T\cap A_l$, nên $T\subset(A_k \cup A_l)$, suy ra $|A_k \cup A_l|\geq |T|$. Mặt khác dựa vào nhận xét ở phần đầu: tồn tại $i,j$ sao cho $1\leq i < j \leq k$ và $H=A_i\cup A_j - \{m\}$ với $m\in S$ nào đó, ta thu được: $|A_k \cup A_l \cup A_i \cup A_j|\geq |T|+(|H|-1) = |T|+(n-|T|-1)\geq n-1>n-2$, mâu thuẫn. Vậy với mọi tập $T'\subset T$ thì $T'$ và $T - T'$ không cùng thuộc $T^*$, hơn nữa $T$ có $2^{|T|}$ tập con nên $|T^*|\leq 2^{|T|-1}$.
Mặt khác, mỗi tập $A_i$ được xác định một cách duy nhất bởi phần giao của nó với $T$ và $H$ cho nên $k \leq |T^*|\cdot |H^*| \leq 2^{|T|+|H|-2} = 2^{n-2}$, kết thúc chứng minh!


Ở chỗ ấy bây giờ thế nào rồi? Thời gian đang khoác chiếc áo thu lên từng khoảng trời Thành phố Hồ Chí Minh, khiến nỗi nhớ ấy cứ đau đáu trong lòng tớ hơn bao giờ hết. Tớ nhớ ấy thật nhiều, người xa xôi ạ!




#210820 Một bài hay...

Đã gửi bởi Mashimaru on 18-08-2009 - 12:30 trong Các bài toán Đại số khác

Anh ilovemath1809 có thể giải thích rõ đa thức đồng biến là gì không ạ? Nếu ý anh đa thức đồng biến là đa thức tăng trên $\mathbb{R}$ thì có thể xây dựng như sau ạ:

Xét đa thức $P(x) = a_nx^n + a_{n-1}x^{n-1} + ... + a_1x + a_0$. Không mất tính tổng quát, có thể giả sử $a_n > 0$. Có hai trường hợp:

+ Nếu $n$ lẻ, ta xây dựng hai đa thức như sau: $A(x) = a_nx^n + a_{n-1}x^{n-1} + ... + (a_1+A)x + a_0$ và $B(x) = Ax$, khi đó rõ ràng $P(x) = A(x) - B(x)$. Hơn nữa $B'(x) = A$ còn $A'(x) = na_nx^{n-1} + (n-1)a_{n-1}x^{n-2} + ... + 2a_2x + a_1+A$. Vì $n$ lẻ nên $\deg{A'}=n-1$ chẵn, hơn nữa ta đã giả sử $a_n >0$ nên tồn tại $\inf{A'(x)}=a$. Chọn $A > |a|$ thì $A'(x) > 0,B'(x) > 0, \forall x\in\mathbb{R}$. Vậy $A(x),B(x)$ là cặp đa thức thỏa điều kiện.

+ Nếu $n$ chẵn, ta xét $A(x) = a_{n+1}x^{n+1} + a_nx^n + ... + (a_1+A)x + a_0$ và $B(x) = a_{n+1}x^{n+1} + Ax$ trong đó $a_{n+1}>0$. Khi đó rõ ràng $P(x) = A(x) - B(x)$, hơn nữa $\deg{A},\deg{B}$ lẻ nên theo trường hợp trên, $A(x)$ và $B(x)$ có thể biểu diễn thành hiệu của hai đa thức đồng biến: $A(x) = A_1(x) - A_2(x),B(x) = B_1(x) - B_2(x)$. Lúc này ta biểu diễn $P(x) = (A_1(x) + B_2(x)) - (A_2(x) + B_1(x))$, với chú ý rằng tổng của hai đa thức đồng biến cũng là một đa thức đồng biến.

Bài toán được giải quyết.



#210158 China TST 2004

Đã gửi bởi Mashimaru on 14-08-2009 - 22:03 trong Số học

Ý của anh Arithmetic đúng rồi ạ. Đó cũng là hướng nghĩ của em. Hơn nữa phần còn lại làm y hệt bài China TST nói trên, chỉ có điều là phần đầu không nặng về kỹ thuật sử dụng cấp và thặng dư bình phương như vậy, nên tuy đó là bài tổng quát hơn nhưng thực ra nếu lấy $a=2$ thì bất đẳng thức của bài China TST vẫn chặt hơn ạ.



#210070 China TST 2004

Đã gửi bởi Mashimaru on 14-08-2009 - 16:28 trong Số học

Một mở rộng của bài toán gốc.

Chứng minh rằng với mọi số nguyên dương $a$ và $p$ là ước nguyên tố của $a$ thì số $a^{p^k}-1$ có ước nguyên tố lớn hơn $kp^k\cdot\dfrac{\ln{p}}{\ln{a}}$



#210015 China TST 2004

Đã gửi bởi Mashimaru on 14-08-2009 - 12:17 trong Số học

cái này làm gì mà theo bđt Bernouli nhỉ


Bất đẳng thức $(1+a)^n \geq 1 + na$ có tên gọi là bất đẳng thức Bernouli mà anh? Em không biết nhớ có đúng không, nhưng dù không phải là bất đẳng thức Bernouli đi nữa thì khai triển nhị thức cũng dễ dàng thấy đúng ạ.

Bài giải trên là đúng rồi ,tuy nhiên có thể đánh giá chặt hơn bdt này : $p(F_n)\geq (4m+9).2^{n+2}+1 $ .Có thể tham khảo lời giải ở quyển này ,page 99 ,tư tuởng giống lời giải của bạn Marshimaru : http://diendantoanho...showtopic=43542


Anh Arithmetic cho em hỏi ký hiệu $p(F_n)$ có ý nghĩa gì ạ? Có phải là có ước nguyên tố của $F_n$ thỏa điều kiện kia không? Hơn nữa, $(4m+9)\cdot 2^{n+2}$ thì $m$ là gì ạ?



#209951 China TST 2004

Đã gửi bởi Mashimaru on 14-08-2009 - 01:56 trong Số học

Không biết có đúng không nữa, chỉ thấy tính toán khủng khiếp. Mọi người check hộ em với ạ.

Với $n=1,2,3$, thì $F_n$ là số nguyên tố, hiển nhiên điều phải chứng minh là đúng. Xét $n\geq 3$. Trước hết ta chứng minh bổ đề sau:

Bổ đề. Nếu $n\geq 3$ và $p$ là ước nguyên tố của $F_n$ thì $p \equiv 1\pmod{2^{n+2}}$.
Thật vậy, đặt $h=ord_p (2)$ thì ta có $2^{2^n}\equiv -1 \pmod p$ nên $2^{2^{n+1}}\equiv 1 \pmod p$, suy ra $h$ chia hết $2^{n+1}$ nhưng không chia hết $2^n$, do vậy $h=2^{n+1}$. Theo tính chất của cấp thì $p\equiv 1\pmod {2^{n+1}}$, mà $n\geq 3$ nên $p\equiv 1\pmod 8$. Từ đó ta có $\left(\dfrac{2}{p}\right)=(-1)^{\dfrac{p^2-1}{8}}=(-1)^{\dfrac{p-1}{8}\cdot (p+1)}=1$ nên tồn tại $H$ sao cho $H^2 \equiv 2\pmod p$. Suy ra $H^{2^{n+1}}\equiv 2^{2^n}\equiv -1\pmod p$. Lặp lại quy trình trên ta thu được $p\equiv 1\pmod {2^{n+2}}$.
Bổ đề được chứng minh.

Vào bài.
Giả sử kết luận của bài toán là sai, tức là các ước nguyên tố của $F_n$ đều không lớn hơn $(n+1)\cdot 2^{n+2}$. Ta xét phân tích tiêu chuẩn $F_n=\prod_{i=1}^{s} (2^{n+2}\cdot k_i +1)^{a_i}$ trong đó $2^{n+2}\cdot k_i +1=p_i$ là các ước nguyên tố của $F_n$. Theo định lý nhị thức thì $(2^{n+2}\cdot k_i +1)^{a_i}=\sum_{t=0}^{a_i} {a_i \choose t} \cdot (2^{n+2}\cdot k_i)^t \equiv 2^{n+2}\cdot a_ik_i +1\pmod {2^{2(n+2)}}$. Suy ra $1+2^{2^n}=\prod_{i=1}^{s} (2^{n+2}\cdot k_i +1)^{a_i}\equiv \prod_{i=1}^{s} (2^{n+2}\cdot a_ik_i +1)\equiv 2^{n+2}\sum_{i=1}^{s}a_ik_i + 1 \pmod {2^{2(n+4)}}$. Từ đây $\sum_{i=1}^{s} a_ik_i\equiv 0\pmod{2^{n+2}}$.
Mặt khác, do $p_i=2^{n+2}\cdot k_i +1\leq (n+1)\cdot2^{n+2}$ nên $k_i\leq n+1$. Theo bất đẳng thức Bernouli ta có: $(1+2^{n+2})^{\dfrac{2^n}{n+2}}>1+2^{2^n}=\prod_{i=1}^{s} (2^{n+2}\cdot k_i +1)^{a_i} \geq (2^{n+2}+1)^{\sum_{i=1}^s a_i}$. Suy ra được $\sum_{i=1}^{s} a_i < \dfrac{2^n}{n+2}$, và từ đây suy ra $\sum_{i=1}^{s} a_ik_i < 2^n\cdot\dfrac{n+1}{n+2}<2^n$. Điều này mâu thuẫn với việc $\sum_{i=1}^{s} a_ik_i\equiv 0\pmod{2^{n+2}}$.
Vậy điều giả sử là sai. Kết thúc chứng minh!

Nhớ ấy quá, tớ không ngủ được. Tặng ấy lời giải này H. nhé ^^



#209844 Một tính chất của đường đối trung

Đã gửi bởi Mashimaru on 13-08-2009 - 17:49 trong Hình học

Cho $\triangle ABC$ có $AA_1$ là đường đối trung với $A_1\in BC$. Gọi $B_a,C_a$ là các điểm trên các cạnh $AC,AB$ sao cho $A_1B_aAC_a$ là hình bình hành. Khi đó, ta dễ thấy $B,C,B_a,C_a$ đồng viên. Gọi $O_a$ là tâm của đường tròn đi qua $B,C,B_a,C_a$. Định nghĩa tương tự các điểm $O_b,O_c$. Chứng minh rằng $AO_a,BO_b,CO_c$ đồng qui tại một điểm $I$, hơn nữa $I$ cách đều hai điểm Brocard của $\triangle ABC$.



#205644 Tin tức về IMO 2009

Đã gửi bởi Mashimaru on 18-07-2009 - 23:47 trong Thi HSG Quốc gia và Quốc tế

bất ngờ quá


Sao lại bất ngờ hả anh?

Còn 3Gold thì khó lắm ạ. Anh Duy và Hùng thì gold rồi, còn em chấp nhập Silver, không hối tiếc ạ >"<



#205643 IMO 2009 P4

Đã gửi bởi Mashimaru on 18-07-2009 - 23:44 trong Thi HSG Quốc gia và Quốc tế

Đây là lời giải của em trong lúc thi, không hiểu sao hôm ấy bị con gì cắn mà không nhìn ra lời giải thuần túy nữa. Ôi...em rất là buồn >.<

Đặt $BC=a,AB=AC=b$, sau đó, "chỉ cần" 3 trang giấy thi với "những biến đổi đơn giản", ta thu được điều kiện cần và đủ để $\widehat{BEK}=45^o$ là:

$\dfrac{2a^2(a-b)^2(a^2-2b^2)}{(a+b)^4(2b-a)^2}\times M(a,b)=0$

Trong đó $M(a,b)$ là đa thức bậc bốn, thuần nhất theo $a,b$ và luôn dương với điều kiện $0<a<2b$.

Từ đó suy ra $\widehat{BAC}=60^o$ hoặc $\widehat{BAC}=90^o$.



#196781 Vietnam TST 2009

Đã gửi bởi Mashimaru on 02-05-2009 - 23:53 trong Thi HSG Quốc gia và Quốc tế

Không khí nóng lên qua từng ngày, lúc nào Hiếu Mashi ra ngoài này anh mời 1 bữa trà đá no say nhá :)


Em chỉ nghe các thầy trường em nói là khoảng giữa tháng 5 thì em phải ra Thủ đô chứ không biết lịch cụ thể là thế nào ạ. Còn cái vụ mà anh nói này thì...anh mời em mới đúng chứ ạ ^o^



#196379 Vietnam TST 2009

Đã gửi bởi Mashimaru on 30-04-2009 - 22:10 trong Thi HSG Quốc gia và Quốc tế

Vâng ạ. Vì có dạo VMF mình down server lâu quá nên lúc mở lại em ko biết ạ. Cảm ơn hai anh vuthanhtu_hd và MrMATH ạ ^_^



#196373 Vietnam TST 2009

Đã gửi bởi Mashimaru on 30-04-2009 - 21:12 trong Thi HSG Quốc gia và Quốc tế

Em ở PTNK TPHCM, xin chào mọi người ạ. Em rất vui vì năm nay may mắn được góp mặt cùng các anh và các bạn. Mong rằng sẽ học hỏi được nhiều cũng như được mọi người giúp đỡ ạ ^_^



#192655 Định Lý Viet

Đã gửi bởi Mashimaru on 21-10-2008 - 22:20 trong Tài nguyên Olympic toán

@ Hiếu: Em hãy thử sử dụng chương trình pdf24.


Thưa thầy, thầy có thể cho em xin llink của chương trình ấy được không ạ?

Em nên dùng Pdf Factory Pro, cái này vừa nhẹ dùng lại hiêu quả, convert cũng không bị lỗi Fonts!! ^^

Đây này em: http://diendantoanho...showtopic=40364


Anh ơi, cái link anh đưa có download được đâu ạ! Nó báo "Board Message..." :D



#192608 Định Lý Viet

Đã gửi bởi Mashimaru on 20-10-2008 - 22:04 trong Tài nguyên Olympic toán

Vậy à ^^
Em là ai vậy :D ( Họ tên ấy ^^ ) anh search cái này thấy trên Google blog


Em tên...đầu tiên ạ :D. Cái blog wordpress ấy cũng của em luôn ^.^

Anh chị nào đã coi thử cái file pdf ấy thì sẽ thấy nó bị lỗi tất cả những chữ N. Cái đó là do em làm từ Word rồi print qua pdf bằng Primo PDF printer. Vậy liệu có chương trình nào chuyển đổi cho chất lượng tốt hơn không ạ???



#192417 Định Lý Viet

Đã gửi bởi Mashimaru on 18-10-2008 - 13:12 trong Tài nguyên Olympic toán

Hờ hờ, bài này của nhóm em... :(



#191687 Buồn ơi buồn đến bao giờ

Đã gửi bởi Mashimaru on 21-09-2008 - 12:23 trong Các dạng toán khác

Vâng, cách giải của em cũng giống anh đấy ạ. Nhưng vấn đề là liệu có tồn tại số nào thỏa điều kiện đề bài đưa ra ko??? Em nghĩ rằng nếu ko có số nào như vậy thì chẳng lẽ những gì chúng ta đang chứng minh có khi nào là vô ích sao ạ???